LSAT and Law School Admissions Forum

Get expert LSAT preparation and law school admissions advice from PowerScore Test Preparation.

 Administrator
PowerScore Staff
  • PowerScore Staff
  • Posts: 8916
  • Joined: Feb 02, 2011
|
#33749
Complete Question Explanation

Flaw in the Reasoning—#%. The correct answer choice is (B)

In this stimulus, the reformer reaches a conclusion based on statistical data, which on the LSAT is a nearly certain sign of a pending logical flaw. The reformer considers the national crime rate, defined as “the annual number of crimes per 100,000 people,” for the past 20 years, and finds that there has been no significant reduction in the crime rate during that period. At the same time, the reformer points out, the percentage of the population that is incarcerated has risen, as has public spending on prisons. Based on this data, the reformer concludes that “putting more people in prison cannot help reduce the crime rate.”

The problem with the reformer’s argument is that it assumes that the crime rate would have remained the same even in the absence of the increased rate of incarceration. However, there is no evidence to support that position. Since the increase in the incarceration rate actually occurred, we cannot know for sure what the national crime rate would have been otherwise. For all we know, without the increase in the rate of incarceration, the national crime rate would have skyrocketed. Since we cannot be certain what effect the increased rate of incarceration had on the crime rate, yet the reformer reaches the definitive conclusion that “putting more people in prison cannot help reduce the crime rate,” this argument is flawed.

The question stem identifies this as a Flaw in the Reasoning question. Our prephrase is that the correct answer choice will describe the reformer’s error in assuming that even in the absence of the increased rate of incarceration, the national crime rate would have stayed the same.

Answer choice (A): This answer choice describes an Error of Division, in which the author assumes that what is true of the whole (i.e., the national crime rate) must be true of each part of the whole (i.e., the crime rate reported by each police precinct). Since the stimulus did not make this error, this answer choice is incorrect.

Answer choice (B): This is the correct answer choice because it relates the reformer’s flawed assumption that the national crime rate would have remained about the same even without the increase in the rate of incarceration.

Answer choice (C): Here, the answer choice is incorrect because it focuses on the number of people in the country, which is irrelevant since the evidence had to do with the national crime rate, a percentage concept.

Answer choice (D): This answer choice is inconsistent with the stimulus. The reformer did not discuss the potential effectiveness of alternatives to imprisonment. Rather, the reformer limited the argument to the position that increased imprisonment cannot reduce crime.

Answer choice (E): In this case, the answer choice comes out of left field, bearing no relationship to the argument in the stimulus. Nothing in the reformer’s argument would indicate an assumption that the number of prisoners must be proportional to the number of crimes committed. In fact, since we only know that there has been no significant reduction in the national crime rate, we cannot even see whether the number of crimes committed has gone up or down. For all we know, the number of crimes has decreased significantly even though the rate itself has not.
 emilysnoddon
  • Posts: 64
  • Joined: Apr 22, 2016
|
#25772
For this question, I was choosing between answer choices B and C. I understand why B is correct but am wondering why C isnt correct. If the overall population increases wouldnt this explain the rise in crime rate?
 Nikki Siclunov
PowerScore Staff
  • PowerScore Staff
  • Posts: 1362
  • Joined: Aug 02, 2011
|
#25895
Hi Emily,

Thanks for your question. To help you better, please provide a more detailed breakdown of how you understood the argument. At the very least, we'd like to see evidence that you were able to:
  • Deconstruct the stimulus into premises/conclusion.
  • Understand whether the conclusion logically follows from the premises, and if not - why not?
  • Correctly identify the type of question in the stem.
  • Prephrase an answer to that question. (Don't be afraid if your prephrase was off - we still need to see what it was :).
  • Defend your choice of (incorrect) answer choice.
The more you tell us about your method of approach, the better we can help you figure out what you did wrong.

Thanks!
 jlam061695
  • Posts: 62
  • Joined: Sep 17, 2016
|
#30119
Like what Emily said, I also had trouble with C. If the population increased significantly over the past 20 like C says, and there was no significant reduction in the crime rate (which could mean that the crime rate could have increased significantly, increased slightly, remained the same, or even decreased slightly), then isn't that factor something that the reformer overlooked? Is C wrong because it does not address the reformer's conclusion about crime rate directly?

I also have problems with B because it says that the reformer "ignores the possibility that the crime rate would have significantly increased," while the issue here is the lack of significant reduction in crime.
 Emily Haney-Caron
PowerScore Staff
  • PowerScore Staff
  • Posts: 577
  • Joined: Jan 12, 2012
|
#30133
Hi jlam,

The crime rate, remember, is the number of crimes per 100,000 people. So, even if the population goes way up, that doesn't necessarily impact the crime rate, because the crime rate is a proportion. The percentage of the population in prison is also a proportion; so even if the population has gone way up, that wouldn't mean the percentage of the population in prison went way up - it might just mean the actual number of people went way up. Proportions/percentages are conflated with real numbers in C, and that's why C is incorrect.

B, on the other hand, is a pretty good answer. B is saying that it could be we'd have way more crimes per 100,000 people than we do now if we hadn't been putting so many people in prison. If that's true, then putting more people in prison actually did reduce crime from what it would otherwise be. Let's say we have 100 crimes per 100,000 people right now. B is saying that, if we hadn't increased the percentage of the population in prison, we might now have 200 crimes per 100,000 people. Putting all those extra people in prison, then, (in this example) cut the crime rate in half from what it would otherwise be!

Does that make sense? Sometimes trying it out with real numbers can help when you get stuck thinking through a question like this!
 jlam061695
  • Posts: 62
  • Joined: Sep 17, 2016
|
#30187
Yes, thank you for the explanation!!
 kcho10
  • Posts: 69
  • Joined: Nov 02, 2015
|
#43179
Hello,

I am confused because the approach for this question seems to be somewhat at odds with the explanation for #15 ("Market Consultant: Last year...").

I understand that #13 is a #% issue, but isn't it also a causal conclusion? For this question I can fully comprehend why B is right, I think. Like the explanations said, it shows that the crime rate would have been even lower if they had not put more people in prison. Isn't that similar to saying that "no effect, no cause?"

So then if that's the case, how can the same type of answer choice be incorrect for #15 in answer choice A? The only possible thing that I can think of is that A is very vague by saying 'lower still' while B for #13 says 'significantly increased'. Is it that for #15, even if there is a very slight dip in effect without the supposed cause, that doesn't necessarily refute that the supposed cause was the actual cause?
I hope this makes sense...Thank you
 Claire Horan
PowerScore Staff
  • PowerScore Staff
  • Posts: 408
  • Joined: Apr 18, 2016
|
#43611
You correctly point out the causal element of this argument: The conclusion is that putting more people in prison does not cause crime reduction. This conclusion that there is no causality assumes that there were no other changes over a 20-year period beyond the ones mentioned in the stimulus. This is a common type of LSAT question--you cannot assume that no other factors influenced a situation besides those mentioned or that no changes occurred over time.

Another issue with Problem #15, answer choice A that you didn't mention is that the alternative is not between the failed advertising campaign and no campaign at all. If the company had listened to the marketing consultant, they presumably would have come up with some other alternative advertising scheme or at least made better use of the money spent in the campaign. An advertising campaign that is "ill-conceived" can still be better than nothing. Answer choice B is better because it more generally points out that there may be no causal connection between the advertising campaign and the poor sales.
 ltowns1
  • Posts: 61
  • Joined: May 16, 2017
|
#48859
I got fooled by (C) because I thought it was a percentage/number problem. Had the stimulus simply said that the rate of crime was increasing, instead of giving a specific rate (per 100,000), would that have made the population relevant? What would have made this into a percentage number problem?
 Adam Tyson
PowerScore Staff
  • PowerScore Staff
  • Posts: 5153
  • Joined: Apr 14, 2011
|
#49088
If the argument had said "The survey shows no significant reduction in the number of crimes in the past 20 years", then this would be a numbers/percentage problem, ltowns1. If that was the argument, then an increase in the population, coupled with a higher percentage of the population in prison and the same number of crimes (so a lower rate of crimes per capita) might actually weaken the argument. Since the author never switched to numbers, but stuck with his percentages evidence only, then the numbers don't really matter!

Get the most out of your LSAT Prep Plus subscription.

Analyze and track your performance with our Testing and Analytics Package.